General Dermatology MCQs

February 13, 2019 | Author: Dr.Tawheed | Category: Systemic Lupus Erythematosus, Sensitivity And Specificity, Psoriasis, Acne Vulgaris, Cancer
Share Embed Donate


Short Description

ETAS MCQ with explanation...

Description

A pregnant woman presents with the following condition which is bothersome to her. Which of the following is the most appropriate treatment option based on FDA classifications of medication in pregnancy? 1

Topical erythromycin/benzoyl peroxide gel

2

Topical tretinoin

3

Tazarotene 0.1% cream

4

Azelaic acid 20% cream

5

Bactrim DS Q/Q(M)-482157 Report a Problem

A pregnant woman presents with the following condition which is bothersome to her. Which of the following is the most appropriate treatment option based on FDA classifications of medication in pregnancy? 4

Azelaic acid 20% cream

This patient has mild-moderate inflammatory acne. Azelaic acid is the only medication listed which falls under category B. The others listed are category C, except for tazarotene, which is category X Q/Q(M)-482157 Report a Problem

Pilomatricomas has been associated with which of the following conditions? 1

Acute myelogenous leukemia

2

Multiple sclerosis

3

Crohns Disease

4

Myotonic dystrophy

5

Pneumothorax 1

Q/Q(M)-477401 Report a Problem

Pilomatricomas has been associated with which of the following conditions? 4

Myotonic dystrophy

Answer D. Pilomatricomas are benign tumors that are derived from hair matrix cells. This tumor presents more frequently in children as a solitary asympotmatic purple red papule or nodule usually on the head and neck. Mutations in beta catenin is associated with the development of these tumors. Multiple pilomatricomas may be a marker for the development of myotonic dystrophy. This rare disorder is AD, and is associated with difficulty relaxing muscles after contraction. In a review of patients with this disorder, patients tended to have mulitple pilomatricomas. Pilomatricomas can also be found with Turner syndrome, Rubinstein-Taybi, and Churg Strauss syndromes. Q/Q(M)-477401 Report a Problem

Lacrimal gland involvement is characteristically seen in patients with: 1

Lymphoma

2

Sarcoid

3

Rosacea

4

Syphilis

5

Pemphigus Q/Q(M)-477156 Report a Problem

Lacrimal gland involvement is characteristically seen in patients with: 2

Sarcoid

Sarcoid is granulomatous autoimmune condition characterized by the formation of non-caseating granulomas which may affect any organ system. Lacrimal gland involvement occurs in 15-28% of patients. Manifestation of lacrimal gland involvement includes bilateral painless swelling. Q/Q(M)-477156 Report a Problem

A patient presents requesting removal of a tattoo. Examination reveals a tattoo with red, orange, yellow, and purple pigment. Which laser would be appropriate to treat this tattoo? 1

Q switched Nd:YAG (532nm)

2

Q switched alexandrite (755nm)

3

Q switched ruby (694nm)

4

Nd:Yag (1064nm)

5

Alexandrite (755nm) Q/Q(M)-482799 Report a Problem

2

A patient presents requesting removal of a tattoo. Examination reveals a tattoo with red, orange, yellow, and purple pigment. Which laser would be appropriate to treat this tattoo? 1

Q switched Nd:YAG (532nm)

The Q switched Nd:YAG can also be used to treat tan pigment. QS alexandrite and QS ruby can remove green pigment. QS Nd:YAG is a good choice for patients with darker skin color. Q/Q(M)-482799 Report a Problem A young man presents with explosive onset of severe cystic acne with acute, suppurative nodules and plaques that ulcerate and form a blackish eschar on the trunk as well as the face. Which of the following is true regarding this entity? 1

Women are more often affected than men

2

P. acnes osteomyelitis presents with lytic changes on x-rays and bone scans

3

The sternoclavicular joint is often involved in this entity

4

Systemic corticosteroids are contraindicated given risk of sepsis

5

High-dose isotretinoin monotherapy is the treatment of choice Q/Q(M)-480164 Report a Problem

A young man presents with explosive onset of severe cystic acne with acute, suppurative nodules and plaques that ulcerate and form a blackish eschar on the trunk as well as the face. Which of the following is true regarding this entity? 3

The sternoclavicular joint is often involved in this entity

Acne fulminans is a rare, explosive form of severe cystic acne affecting young males. Patients may be systemically ill, with leukocytosis, fever, arthralgias, and myalgias. Lytic changes, indicative of a sterile osteomyelitis, can be seen on x-ray and bone scans. The sternoclavicular joint and chest wall are most frequently affected. Treatment is with oral prednisone, intralesional steroids, antibiotics, and isotretinoin. Q/Q(M)-480164 Report a Problem

Which of the following is true regarding acquired C1 esterase inhibitor deficiency? 1

This condition generally occurs in the first or second decade of life

2

Serum C1q is normal

3

C2 and C4 are both decreased

4

C1 esterase inhibitor may be at normal levels with functional impairment

5

Positive family history is common Q/Q(M)-480508 Report a Problem

Which of the following is true regarding acquired C1 esterase inhibitor deficiency? 3

C2 and C4 are both decreased

C1 esterase inhibitor is a protease inhibitor that inhibits the catalytic subunits of the first components of 3

the classical pathway. In the absence of C1 esterase inhibitor, activated C1 and plasmin generate activated C2 kinin, which mediates angioedema. Acquired C1 esterase inhibitor deficiency generally affects adults or elderly individuals with no family history. Serum C1q is decreased. It occurs in the setting of lymphoproliferative disease or rheumatologic illness, where idiotype/anti-idiotype immune complexes consume available C1q and functionally and quantitatively lower the amounts of C1 esterase inhibitor. It can also occur in the setting of autoimmunity directed against the C1 esterase protein. Inherited C1 esterase inhibitor deficiency is detected in the first or second decade of life and is autosomal dominantly inherited. Serum C1q is normal in the inherited form, but there is a defect in the synthesis and/or function of C1 esterase inhibitor. In both the inherited and acquired forms levels of C2 and C4 are decreased because of the uncontrolled actions of C1s. Q/Q(M)-480508 Report a Problem A 2 year-old boy with crusted skin papules is found to have osteolytic defects and diabetes insipidus. These features are seen in: 1

Osteogenesis imperfecta

2

Conradi-Hunnerman disease

3

Metastatic small cell lung carcinoma

4

Hand-Schuller-Christian disease

5

Epidermal nevus syndrome Q/Q(M)-477165 Report a Problem

A 2 year-old boy with crusted skin papules is found to have osteolytic defects and diabetes insipidus. These features are seen in: 4

Hand-Schuller-Christian disease

Hand-Schuller-Christian disease is a chronic multifocal form of Langerhans cell histiocytosis.70% of cases occur between the ages of 2 and 6. The four characteristic clinical findings are bone lesions, diabetes insipidus, exophthalmus, mucocutaneous lesions. Bones lesions are osteolytic and preferentially involve the calvarium. Q/Q(M)-477165 Report a Problem Probiotics, which are cultures of potentially beneficial gut microflora bacteria, have been studied in the primary prevention of which of the following diseases? 1

Celiac disease

2

Atopic dermatitis

3

Psoriasis

4

Cutaneous T-cell lymphoma

5

Asthma Q/Q(M)-474298 Report a Problem

4

Probiotics, which are cultures of potentially beneficial gut microflora bacteria, have been studied in the primary prevention of which of the following diseases? 2

Atopic dermatitis

Probiotics have been studied in the primary prevention of atopic dermatitis by Kalliomaki et al. Lactobacillus GG cultures were given to pregnant women with a history of atopy to assess the effect of potentially beneficial gut flora on the prevention of atopic disease in their children. The frequency of atopic dermatitis in the children in the probiotic group was half that in the placebo group at two years of life. Q/Q(M)-474298 Report a Problem

Mutations in which of the following genes has been implicated in the pathogenesis of disseminated superficial actinic porokeratosis? 1

BRAF

2

K-RAS

3

PTCH

4

PTEN

5

SART3 Q/Q(M)-482839 Report a Problem

Mutations in which of the following genes has been implicated in the pathogenesis of disseminated superficial actinic porokeratosis? 5

SART3

Disseminated superficial actinic porokeratosis (DSAP) is a disorder characterized by numerous keratotic macules and papules often localized in a photodistribution. Clinically, they are distinguished by a rim of keratotic scale and often may have an atropic appearance centrally. Histologically, they often display a lichenoid infiltrate flanked by characteristic cornoid lamella (inward-bending tiers of parakeratosis with underlying hypogranulosis and dyskeratosis). Recently, mutations in SART3 (squamous cell carcinoma antigen recognized by T-cells 3) have been implicated in a Taiwainese patient cohort affected by DSAP. Of note, while all varieties of porokeratoses have the potential for malignant degeneration and development of squamous cell carcinoma, lesions of DSAP have the lowest risk. Q/Q(M)-482839 Report a Problem

Which contact allergen is found in Krazy glue? 1

Formaldehyde

2

Lanolin alchohol

3

Ethyl cyanoacrylate

4

Octyl-dimethyl-paba

5

Mercaptobenzothiazole Q/Q(M)-476071 Report a Problem

5

Which contact allergen is found in Krazy glue? 3

Ethyl cyanoacrylate

Ethyl cyanoacrylate is a chemical found in Krazy Glue. It can also be found in nail adhesives causing a dermatitis on the face and fingers, nail dystrophy, and a generalized rash. Formaldehyde is a widely used chemical found in a variety of applications and is a common allergen. It can be found in paper, skin and hair products, cosmetics, and permanent press textiles. Lanolin alcohol is found in wool fat, wool wax, adhesives, cosmetics and pharmaceuticals. Octyl-dimethyl-paba is a chemical found in sunscreens and is the most frequently used PABA group sunscreen. Mercaptobenzothiazole is a rubber accelerant and is the most common allergen found in dermatitis to shoes. It is also found in veterinarian products such as flea and tick sprays and powders. Mercapotbenzothiazole can also be found in cutting oil, antifreeze, fungicides and photographic film emulsions. Q/Q(M)-476071 Report a Problem

In the setting of the lupus erythematosus-associated complement deficiency syndrome, which of the following would most often be observed? 1

low C3 and C4

2

low C2 and C3

3

low C2 and C4

4

low C2 only

5

low C3 only Q/Q(M)-480514 Report a Problem

In the setting of the lupus erythematosus-associated complement deficiency syndrome, which of the following would most often be observed? 3

low C2 and C4

In the complement deficiency syndrome, low C2 and C4 are most commonly seen. Photosensitivity, annular SCLE lesions, and Ro antibody formation are commonly observed. Q/Q(M)-480514 Report a Problem

Which of the following is a feature of Schnitzler's Syndrome? 1

Monoclonal IgG gammopathy

2

Thromboembolic events

3

Bronchospasm

4

Hematuria

5

Sensorimotor neuropathy Q/Q(M)-480534 Report a Problem

Which of the following is a feature of Schnitzler's Syndrome? 6

5

Sensorimotor neuropathy

Schnitzler's syndrome presents as episodes of urticarial vasculitis that occur in association with a monoclonal IgM M component. Fever, lymphadenopathy, hepatosplenomegaly, bone pain, and sensorimotor neuropathy also occur. Q/Q(M)-480534 Report a Problem Which of the following is a manifestation of psoriasis of the nail matrix? 1

Splinter hemorrhages

2

�Oil spots�

3

Subungual hyperkeratosis

4

Pits

5

Onycholysis Q/Q(M)-474319 Report a Problem

Which of the following is a manifestation of psoriasis of the nail matrix? 4

Pits

Psoriatic nail changes may be of nail matrix or nail bed origin. Pits are the common finding; splinter hemorrhages the least. Psoriatic nail changes of matrix origin include: pits (representing focal psoriasis of the proximal matrix) and leukonychia. Psoriatic nails changes of nail bed origin include: salmon spots, �oil spots,� onycholysis, subungual hyperkeratosis, and splinter hemorrhages. Q/Q(M)-474319 Report a Problem Which of the following leukodermas has a normal number of epidermal melanocytes? 1

Albinism

2

Vitiligo

3

Piebaldism

4

Waardenburg's syndrome

5

Ziprowski-Margolis syndrome Q/Q(M)-480477 Report a Problem

Which of the following leukodermas has a normal number of epidermal melanocytes? 1

Albinism

Patients with albinism have a normal number of epidermal melanocytes, however, these melanocytes synthesize inadequate amounts of melanin. Vitiligo, piebaldism, Waardenburg's syndrome and Ziprowski-Margolis syndrome all feature a decreased number or total absence of epidermal melanocytes. Q/Q(M)-480477 Report a Problem After being treated for several months with doxycycline, this patient develops a gram negative folliculitis. What is the next appropriate therapy? 7

1

Tetracycline

2

Bactrim

3

Isotretinoin

4

Ceftriaxone

5

Cefepime Q/Q(M)-476722 Report a Problem

After being treated for several months with doxycycline, this patient develops a gram negative folliculitis. What is the next appropriate therapy? 3

Isotretinoin

Gram negative folliculitis may occur after prolonged antibiotic therapy for acne vulgaris. It should be suspected in patients who are well controlled and then suddenly flare. The treatment of choice is isotretinoin. Q/Q(M)-476722 Report a Problem

You are consulted to evaluate this patient with tender, warm plaques on his shins. What other physical signs should you look for? 1

Exophthalmos

2

Uveitis

3

Tachycardia

4

Clubbing of the fingers 8

5

Cough Q/Q(M)-480561 Report a Problem

You are consulted to evaluate this patient with tender, warm plaques on his shins. What other physical signs should you look for? 2

Uveitis

Erythema Nodosum (EN) is represented by tender, warm,nodules and plaques, often but not exclusivley located on the anterior shin. It represents a reactive panniculitis. Causes include, infections (Streptococcal, tuberculosis, yersina, mycoplasma, campylobacter,salmonella, histoplasmosis, blastomycosis, coccidiomycosis), drugs (sulfonamides, gold, and OCP's), enteropathies, pregnancy, hodgkin's disease and lymphoma and sarcoidosis. Loefgren's disease is a varient of sarcoidosis with EN, hilar adenopathy, fever, uveitis and arthritis. Differential diagnosis includes: pretibial myxedema and erythema induratum. Q/Q(M)-480561 Report a Problem

TH2 immune responses: 1

Are associated with cell-mediated immunity

2

Produce IL-6

3

Produce IFN-gamma

4

Produce TNF-beta

5

Produce IL-2 Q/Q(M)-480484 Report a Problem

TH2 immune responses: 2

Produce IL-6

TH1 cells produce IL-2, IFN-gamma, and TNF-beta, and are associated with cell-mediated immunity. TH2 cells produce IL-4, IL-5, IL-6, IL-10, and IL-13, and are associated with antibody-mediated immune responses. Q/Q(M)-480484 Report a Problem

Monotherapy for acne with topical antibiotics is discouraged because of: 9

1

Slow onset of comedolytic action

2

Potential for irritation

3

Lack of anti-inflammatory action

4

Potential for bacterial resistance

5

Poor patient compliance Q/Q(M)-474321 Report a Problem

Monotherapy for acne with topical antibiotics is discouraged because of: 4

Potential for bacterial resistance

Topical antibiotics reduce the population of P. acnes on the skin, and thus are indirectly antiinflammatory. In contrast to topical retinoids, topical antibiotics are not comedolytic. They are generally well tolerated by patients. Q/Q(M)-474321 Report a Problem The first site in body that shows yellowish pigmentation in carotenoderma is 1

Face

2

Trunk

3

Palms and soles

4

Nails

5

Sclera Q/Q(M)-482111 Report a Problem

The first site in body that shows yellowish pigmentation in carotenoderma is 1

Face

Carotenoderma is yellowish discoloration of the skin secondary to carotenemia. Carotene is excreted by sebaceous glands and in sweat, so the yellow pigmentation appears first on the face (especially nasolabial folds and forehead) and then becomes diffusely distributed with accentuation in palms and soles. In contrast to jaundice, carotenoderma spares mucous membranes and sclera Q/Q(M)-482111 Report a Problem In addition to the lesions seen on the feet of this young man he also had geographic tongue, erythematous plaques on his penis and arthritis, what HLA type is associated with this syndrome? 1

HLA-B27

2

HLA-B51

3

HLA-DR1

4

HLA-DR3

5

HLA-DQW2 10

Q/Q(M)-482070 Report a Problem

In addition to the lesions seen on the feet of this young man he also had geographic tongue, erythematous plaques on his penis and arthritis, what HLA type is associated with this syndrome? 1

HLA-B27

Reiter syndrome, now referred to as reactive arthritis (ReA), is a condition that most often occurs following enteric or urogenital infections. Reactive arthritis is associated with human leukocyte antigen (HLA)–B27, although HLA-B27 is not always present in individuals who are HIV+. Bacteria associated with reactive arthritis are generally enteric or venereal and include the following: Shigella flexneri, Salmonella typhimurium, Salmonella enteritidis, Streptococcus viridans, Mycoplasma pneumonia, Cyclospora, Chlamydia trachomatis, Yersinia enterocolitica, and Yersinia pseudotuberculosis. Q/Q(M)-482070 Report a Problem

What is the most common variant of morphea in children? 1

Plaque

2

Generalized

3

Bullous

4

Deep (morphea profunda)

5

Linear Q/Q(M)-482550 Report a Problem

What is the most common variant of morphea in children? 5

Linear

Linear morphea is the most common presentation in children, comprising between 40% to 70% of children with morphea. This subtype includes linear morphea of the extremity, en coup de sabre, or progressive facial hemiatrophy, all of which may be accompanied by underlying tissue atrophy. Q/Q(M)-482550 Report a Problem

Presence of which of the following autoantibodies is diagnostic of SLE and not reported in patients with other connective tissue diseases? 1

anti-U1RNP 11

2

anti-dsDNA

3

anti-Ro

4

anti-La

5

anti-Sm Q/Q(M)-480522 Report a Problem

Presence of which of the following autoantibodies is diagnostic of SLE and not reported in patients with other connective tissue diseases? 5

anti-Sm

Anti-Sm is diagnostic of SLE and not reported in patients with other connective tissue diseases. It is found in 15-40% of patients with SLE. Most patients with anti-Sm also have antibodies to U1RNP, but the converse is not true. anti-U1RNP is found in 100% of patients with MCTD and in 30% of patients with SLE (the majority of patients with positive U1RNP have SLE rather than MCTD). Anti-dsDNA correlates with renal involvement in SLE. Anti-Ro and anti-La antibodies are found in LE and Sjogren's, and strongly associated with photosensitivity. Q/Q(M)-480522 Report a Problem

Perifolicular depigmentation is a characteristic cutaneous manifestation of: 1

Dermatomyositis

2

Secondary syphilis

3

Scleroderma

4

Discoid lupus

5

Sarcoidosis

Q/Q(M)-477175 Report a Problem Perifolicular depigmentation is a characteristic cutaneous manifestation of: 3

Scleroderma

Scleroderma is a systemic disease which may affect almost any organ in the body. Cutaneous findings are characterized by symmetric, thickening of the skin. Dyspigmentation may occur with �salt and pepper� appearance due to perifollicular depigmentation. Q/Q(M)-477175 Report a Problem Which of the following hormones bind the androgen receptor? 1

Dehydroepiandrosterone

2

Androstenedione

3

Dihydrotestosterone

4

Dehydroepiandrosterone and dihydrotestosterone

5

Dehydroepiandrosterone, androstenedione, and dihydrotestosterone 12

Q/Q(M)-480496 Report a Problem

Which of the following hormones bind the androgen receptor? 3

Dihydrotestosterone

Only testosterone and dihydrotestosterone bind the androgen receptor, thus adrenal androgens (androstenedione and dehydroepiandrosterone) virilize only in so far as they serve as precursors for testosterone and dihydrotestosterone. Q/Q(M)-480496 Report a Problem A patient presents with hemorrhagic onycholysis. The drug class most commonly associated with this finding is: 1

Quinolone antibiotics

2

Systemic retinoids

3

Calcineurin inhibitors

4

Taxanes

5

Tetracyclines Q/Q(M)-482324 Report a Problem

A patient presents with hemorrhagic onycholysis. The drug class most commonly associated with this finding is: 4

Taxanes

Taxane probably cause nail changes more commonly than other drugs. Cutaneous toxicity has been reported with taxanes and includes erythema and desquamation, involving primarily the hands. Taxanes exert their cytotoxic effect by reversibly binding the β-subunit of tubulin, thereby inducing tubulin polymerization and inhibiting microtubule depolymerization. A balance between polymerization and depolymerization is needed for normal microtubule function. Taxanes disrupt this balance, leading to arrest at the G2/M phase of the cell cycle. Q/Q(M)-482324 Report a Problem

Diarrhea, Dementia and a photosensitive dermatitis are associated with a deficiency of which vitamin? 1

Niacin

2

Biotin

3

Thiamine

4

Riboflavin

5

Pyridoxine Q/Q(M)-480472 Report a Problem

13

Diarrhea, Dementia and a photosensitive dermatitis are associated with a deficiency of which vitamin? 1

Niacin

Niacin (Vitamin B3) deficiency is associated with a photosensitive dermatitis, diarrhea and dimentia. The photosensitive dermatitis classically involves the face, neck and upper chest (Casal's necklace) and forearms (as pictured). Other findings may include angular cheilitis and thickening and hyperpigmentation of skin overlying bony prominences. Q/Q(M)-480472 Report a Problem

What is the most common presentation of psoriatic arthritis? 1

Asymmetric oligo- or polyarthritis

2

Symmetric polyarthritis

3

Spondylitis (axial)

4

Distal interphalangeal joint (DIP) disease

5

Arthritis mutilans Q/Q(M)-480461 Report a Problem

What is the most common presentation of psoriatic arthritis? 1

Asymmetric oligo- or polyarthritis

Psoriatic arthritis affects 20-40% of patients with psoriasis. Most (~80%) of psoriatic arthritis patients are rheumatoid factor negative. All of the answer choices represent possible presentations of psoriatic arthritis, but asymmetric oligo- or polyarthritis is the most common presentation. Q/Q(M)-480461 Report a Problem 14

In this patient with an autoimmune disorder, which autoantibody would be indicative of increased risk of pulmonary disease? 1

U1RNP

2

DsDNA

3

anti-Jo-1 antibody

4

Mi-2 antibody

5

Scl-70 antibody Q/Q(M)-476825 Report a Problem

In this patient with an autoimmune disorder, which autoantibody would be indicative of increased risk of pulmonary disease? 3

anti-Jo-1 antibody

The patient depicted has dermatomyositis. Autoantibodies to anti-Jo-1 antibody targets histidyl transfer RNA synthetase. In dermatomyositis correlates with the development of pulmonary disease. Q/Q(M)-476825 Report a Problem

All of the following drugs have been reported to cause rash similar to the attached image except 1

Hydroxyurea

2

D-penicillamine

3

Statins

4

Phenytoin

5

Captopril Q/Q(M)-482135 Report a Problem

15

All of the following drugs have been reported to cause rash similar to the attached image except 5

Captopril

The image shows symmetrical erthematous to violaceous plaque on dorsal hands with some periungual erythema and telangiectasia. Skin biopsy was consistent with interface dermatitis and mucin. Many drugs have been reported to cause dermatomyositis-like picture, which include hydroxyurea, D-penicillamine, statins, phenytoin and alfuzosin (alpha antagonist for BPH). Captopril is not associated with DM-like rash. Q/Q(M)-482135 Report a Problem

This woman has restricted ability to open her mouth with tight bound down skin of her hands and pulmonary fibrosis. In addition to a positive ANA with a nucleolar pattern, what is the most common autoimmune antibody that is associated with pulmonary fibrosis? 1

Topoisomerase I antibodies (formerly Scl-70)

2

Fibrillarin antibodies

3

Anti-U3RNP antibodies

4

Anti-PM-Scl antibodies

5

Anti-thyroglobulin antibodies Q/Q(M)-482066 Report a Problem

This woman has restricted ability to open her mouth with tight bound down skin of her hands and pulmonary fibrosis. In addition to a positive ANA with a nucleolar pattern, what is the most common autoimmune antibody that is associated with pulmonary fibrosis? 16

1

Topoisomerase I antibodies (formerly Scl-70)

This woman has scleroderma which is a systemic disease characterized by skin induration and thickening. The cutaneous findings are accompanied by various degrees of tissue fibrosis and chronic inflammatory infiltration in numerous visceral organs, prominent fibroproliferative vasculopathy, and humoral and cellular immune alterations. Antinuclear antibodies are present in about 95% of the patients, usually with a speckled or homogenous pattern. A nucleolar pattern, although less common, is more specific for systemic sclerosis. Topoisomerase I antibodies (formerly Scl-70) are present in approximately 30% of patients with diffuse disease (absent in limited disease) and are associated with pulmonary fibrosis. Anticentromere antibodies are present in about 60-90% of patients with limited disease and are rare in patients with diffuse disease. Fibrillarin antibodies and antibodies to U3 ribonucleoprotein (RNP) may also be present but are more common in patients with skeletal muscle involvement. Anti-U3RNP is present mostly in patients with diffuse disease with overlap syndromes. Anti-ThRNP is present mostly in limited disease and is associated with more extensive visceral disease. Anti-PM-Scl is present in limited and overlap states and is associated with myositis and renal involvement. Q/Q(M)-482066 Report a Problem

Increased chylomicrons are a feature of which type of hyperlipoproteinemia? 1

Type I

2

Type IIa

3

Type IIb

4

Type III

5

Type IV Q/Q(M)-480549 Report a Problem

Increased chylomicrons are a feature of which type of hyperlipoproteinemia? 1

Type I

Type I (familial lipoprotein lipase deficiency or apoprotein CII deficiency) hyperlipoproteinemia has increased chylomicrons as a feature, and presents with eruptive xanthomas and lipemia retinalis. For the other hyperlipoproteinemias, the associated lipid abnormalities are as follows: Type IIa -- increased LDL; Type IIb -- increased LDL and VLDL; Type III -- increased IDL; Type IV -- increased VLDL. Type V has increased chylomicrons and increased VLDL. Q/Q(M)-480549 Report a Problem

A patient presents with cutaneous lesions suggestive of sarcoidosis. You consider ordering a serum ACE level to help with the diagnosis. What is the sensitivity and specificity of checking an ace level in this pt? 1

Sensitivity: 60 Specificity: 80

2

Sensitivity: 90 Specificity: 60

3

Sensitivity: 95 Specificity: 80

4

Sensitivity: 60 Specificity: 60 17

5

Sensitivity: 90 Specificity: 25 Q/Q(M)-482232 Report a Problem

A patient presents with cutaneous lesions suggestive of sarcoidosis. You consider ordering a serum ACE level to help with the diagnosis. What is the sensitivity and specificity of checking an ace level in this pt? 1

Sensitivity: 60 Specificity: 80

Serum ACE levels are positive in only approximately 60% of patients with sarcoidosis. However the specificity is only 80%. Therefore, serum ace levels are less helpful in diagnosis (they can be helpful in following treatment response.) For example, if your pretest clinical suspicion of sarcoidosis is 25% (i.e. 1 of 4 likely possible etiologies based on differential diagnosis), then of a 1000 patients tested, 250 will have sarcoidosis. 160 of these will be detected by an abnormal serum ace level, but 150 of the nonsarcoidosis patients will also have an abnormal ace level giving you a positive predictive value of just over 50% making it a poor diagnositic test. Q/Q(M)-482232 Report a Problem

Pyostomatitis vegetans is characteristically associated with which systemic disease? 1

Pemphigus vulgaris

2

Ulcerative colitis

3

Lichen planus

4

Lymphoma

5

Rheumatoid arthritis Q/Q(M)-477349 Report a Problem

Pyostomatitis vegetans is characteristically associated with which systemic disease? 2

Ulcerative colitis

Pyostomatitis vegetans is a pustular, vegetative variant of pyoderma gangrenosum, found in the oral mucous membranes. It is most frequently associated with inflammatory bowel disease. Q/Q(M)-477349 Report a Problem

Best treatment option for this stable type of vitiligo is 1

Phototherapy with narrow band UVB

2

Excimer laser

3

Oral prednisone

4

20% monobenzyl ether of hydroquinone

5

Nitrogen mustard Q/Q(M)-482122 Report a Problem

18

Best treatment option for this stable type of vitiligo is 4

20% monobenzyl ether of hydroquinone

The picture shows generalized type of vitiligo or vitiligo universalis. Patients who have widespread disease with only a few areas of normally pigmented skin in exposed sites can be treated with depigmenting agents. The patients must be carefully chosen, i.e. adults who recognize that their appearance will be altered significantly and who understand that depigmentation requires lifelong care of the skin (sunscreens, protective clothing, etc.). The most commonly used agent is monobenzyl ether of hydroquinone (MBEH) 20% applied twice daily to the affected areas for 9-12 months or longer. Monobenzyl ether of hydroquinone is a potent irritant and/or allergen, and an open use test should be performed before more widespread application. It normally takes 1-3 months to initiate a response, and a loss of pigment can occur at distant sites. Although depigmentation from MBEH is considered permanent, repigmentation (especially perifollicular) can be seen following a sunburn or even intense sun exposure. Monomethyl ether of hydroquinone in a 20% cream can be used as an alternative to MBEH. Side effects include contact dermatitis, exogenous ochronosis and leukomelanoderma en confetti. Phototherapy and excimer laser are not good or practical choices for this type of vitiligo. Nitrogen mustard is not used in vitiligo. Q/Q(M)-482122 Report a Problem

What is the most common malignancy associated with this condition in this female? 1

Brain cancer

2

Thyroid cancer

3

Breast cancer

4

Ovarian cancer

5

Liver cancer Q/Q(M)-476818 Report a Problem 19

What is the most common malignancy associated with this condition in this female? 4

Ovarian cancer

Dermatomyositis is an autoimmune polymyositis with characteristic cutaneous findings. Patients with dermatomyositis should be screened for a underlying malignancy. Risk factors for having an underlying malignancy include a negative ANA, adult age, and female gender. Ovarian cancer is one of the most frequently associated dermatomyositis. Q/Q(M)-476818 Report a Problem

What is the treatment of choice for this condition? 1

Nystatin

2

Fluconazole

3

Better oral hygiene

4

Penicillin

5

Acyclovir Q/Q(M)-476814 Report a Problem

What is the treatment of choice for this condition? 3

Better oral hygiene

Black tongue is associated with poor oral hygiene, the use of medications, and radiation to the head and 20

neck region. In many cases, simply brushing the tongue with a toothbrush or using a commercially available tongue scraper is sufficient improve the condition. Q/Q(M)-476814 Report a Problem

Patients with Dermatitis Herpetiformis are most likely to have: 1

Antibodies to BPAg2

2

Antibodies to transglutaminase 3

3

Mutations in transglutaminase I

4

Mutations in laminin 5

5

Mutations in plectin Q/Q(M)-482146 Report a Problem

Patients with Dermatitis Herpetiformis are most likely to have: 2

Antibodies to transglutaminase 3

In dermatitis herpetiformis, antibodies are found to transglutaminase 3, and the direct immunofluorescent studies show granular IgA and C3 in the dermal papillae. Antibodies to BPag2 are found in bullous pemphigoid. Mutations in plectin are found in EBS with muscular dystrophy. Mutations in laminin 5 are found in patients with JEB,Herlitz type. Mutations in transglutaminase I are found in pateints with lamellar ichthyosis and non bullous congenital ichthyosiform erythroderma. Q/Q(M)-482146 Report a Problem

Histologic examination of this chronic pruritic plaque on the back, which of the following stains would NOT be positive? 1

Von Kossa

2

Cotton dyes

3

Crystal violet

4

PAS

5

Thioflavin T 21

Q/Q(M)-476863 Report a Problem

Histologic examination of this chronic pruritic plaque on the back, which of the following stains would NOT be positive? 1

Von Kossa

Macular amyloid is a form of keratin derived amyloid, which is typically located between the shoulder blades. Many stains are used to identify amyloid in the skin including Congo red, cotton dyes, crystal violet, PAS and thioflavin T. Q/Q(M)-476863 Report a Problem

The differential diagnosis for the attached image should include all of the followings except 1

Candidiasis

2

Lichen planus

3

Contact dermatitis

4

Inverse psoriasis

5

Erythrasma Q/Q(M)-482121 Report a Problem

The differential diagnosis for the attached image should include all of the followings except 22

5

Erythrasma

The image shown is inverse type of psoriasis. Although psoriatic lesions are classically distributed on the extensor surfaces (the elbows, knees, and lumbosacral regions), lesions may also be found in a flexural distribution with involvement of the axillae, groin, perineum, central chest, and umbilical region. Differential diagnosis include all other choices in question. Although erythrasma might be included in differentials in case of groin involvement only, it is unlikly for eryhrasma to presnt on penis with well demarctaed erythamtos plaques. Erythrasma is a superficial bacterial infection of the skin caused by C. minutissimum. It is characterized by asymptomatic, well-demarcated, reddish brown, slightly scaly patches in the groin, axillae, gluteal crease, or inframammary regions, and less often the interdigital spaces of the feet. Erythrasma is frequently confused with a dermatophyte infection, it can be differentiated from tinea infection by the characteristic coral red fluorescence seen when viewed under Wood's lamp illumination (due to the production of porphyrins by the corynebacteria. Q/Q(M)-482121 Report a Problem

This 10 year-old girl presents to your office. These lesions have been present for months. She denies any other systemic complaints. What will you tell her parents? 1

They should expect her to get more lesions in non sun-exposed areas

2

She is more likely to develop systemic lupus erythematosus than an adult with these lesions

3

She is less likely than an adult to develop renal disease

4

She is more likely than an adult to develop cardiovascular disease

5

No hematologic labs are required for evaluation Q/Q(M)-480560 Report a Problem

This 10 year-old girl presents to your office. These lesions have been present for months. She denies any other systemic complaints. What will you tell her parents? 1

They should expect her to get more lesions in non sun-exposed areas

2

She is more likely to develop systemic lupus erythematosus than an adult with these lesions

3

She is less likely than an adult to develop renal disease

4

She is more likely than an adult to develop cardiovascular disease 23

5

No hematologic labs are required for evaluation Q/Q(M)-480560 Report a Problem

This 10 year-old girl presents to your office. These lesions have been present for months. She denies any other systemic complaints. What will you tell her parents? 2

She is more likely to develop systemic lupus erythematosus than an adult with these lesions

Discoid lupus erythematosus (DLE) presents with plaques characterized by scarring, atrophy, follicular plugging, and scale and photosensitivity. Children presenting with DLE have a higher incident of developing systemic lupus (SLE) than adults. Because of progression from DLE to SLE, children should be screened and followed with antinuclear antibodies and anti- DNA antibodies. Children and adolescents have a higher incidence of renal involvement. Treatment for DLE includes topical steroids, oral steroids, and hydroxychloroquine. Q/Q(M)-480560 Report a Problem

You are suspicious for a new diagnosis of lupus in a patient recently treated with penicillamine. Which antibodies would you expect to have been induced by this medication? 1

anti-dsDNA

2

anti-histone

3

anti-SSA

4

anti-SSB

5

anti-Mi-2 Q/Q(M)-480516 Report a Problem

You are suspicious for a new diagnosis of lupus in a patient recently treated with penicillamine. Which antibodies would you expect to have been induced by this medication? 1

anti-dsDNA

Penicillamine induces native systemic lupus erythematosus, associated with anti-dsDNA antibodies, in contrast to drug-induced lupus erythematosus which has been associated with exposure to hydralazine, procainamide, sulfonamides, penicillin, anticonvulsants, minocycline, and INH and is associated with anti-histone antibodies. 24

Q/Q(M)-480516 Report a Problem What autoimmune disease is associated with a PRP-like eruption? 1

Systemic lupus

2

Rheumatoid arthritis

3

Dermatomyositis

4

Scleroderma

5

Pemphigus erythematosus Q/Q(M)-482872 Report a Problem

What autoimmune disease is associated with a PRP-like eruption? 3

Dermatomyositis

The Wong type of dermatomyositis is characterized by erythematous, hyperkeratotic, follicular papules . It can have a limited or generalized distribution. It can evolve into full-blown dermatomyositis with classic cutaneous features. This rare variant bears a striking clinical resemblance to PRP. It can be distinguished on skin biopsy. It has been reported in both adults and children. Some reports suggest the follicular papules are confined to the skin overlying the extensor knees and elbows and this variant may be more common in Asian patients. Q/Q(M)-482872 Report a Problem What is the treatment of choice for neurotic excoriations? 1

Risperidone

2

Doxepin

3

Olanzapine

4

Gabapentin

5

Diphenhydramine Q/Q(M)-482564 Report a Problem

What is the treatment of choice for neurotic excoriations? 2

Doxepin

Doxepin is the treatment of choice for patients with neurotic excoriations. It has both antidepressant and antipruritic effects. Combined psychiatric and pharmacologic intervention is recommended. Other agents such as risperidone, olanzapine, gabapentin, and diphenhydramine are not drugs of choice for this disorder. Q/Q(M)-482564 Report a Problem

The "flag sign" is seen in which of the following conditions? 25

1

psoriasis

2

sarcoidosis

3

hidradenitis suppurativa

4

kwashiorkor

5

telogen effluvium Q/Q(M)-480485 Report a Problem

The "flag sign" is seen in which of the following conditions? 4

kwashiorkor

The "flag sign" is a striking physical finding most readily seen in long and dark hair. Hair that has grown during periods of inadequate nutrition is pale so alternating bands of light and dark can be seen along a single strand (reflecting periods of adequate and inadequate nutrition). The "flag sign" can be seen with other nutritional disorders as well. Q/Q(M)-480485 Report a Problem

A young man treated with minocycline developed blue-black discoloration with acne scars at his cheeks. A Perls stain would show: 1

Increased melanin at the basal layer of the epidermis

2

Black staining granules within macrophages

3

Blue staining granules within macrophages

4

�Muddy brown� pigment granules

5

Giant melanosomes

Q/Q(M)-474305 Report a Problem A young man treated with minocycline developed blue-black discoloration with acne scars at his cheeks. A Perls stain would show: 3

Blue staining granules within macrophages

Perls stain stains hemosiderin (iron) blue. Fontana Masson stains melanin black. Minocycline hyperpigmentation often shows positive staining for both iron and melanin. Q/Q(M)-474305 Report a Problem

Which of the following is true regarding topical therapies for psoriasis? 1

Vitamin D3 analogues deactivate salicylic acid

2

Retinoids are effective in decreasing lesional erythema

3

Anthralin can cause irreversible staining of peri-lesional skin

4

Calcipotriol is deactivated by UV light 26

5

Anthralin inhibits PMNs and monocytes Q/Q(M)-480389 Report a Problem

Which of the following is true regarding topical therapies for psoriasis? 5

Anthralin inhibits PMNs and monocytes

Anthralin, in addition to possessing antiproliferative activity on human keratinocytes, has strong antiinflammatory effects by inhibiting PMNs and monocytes. Vitamin D3 analogues are inactivated by salicylic acid and should be used after UV light (calcipotriol absorbs UV). Retinoids reduce scaling and plaque thickness, but do not generally decrease lesional erythema. Anthralin can stain hair purple and cause reversible brownish discoloration of surrounding skin. Q/Q(M)-480389 Report a Problem What is the diagnosis? 1

Psoriasis

2

Lichen planus

3

Balanitis circinata

4

Candida

5

Syphilis Q/Q(M)-476810 Report a Problem

What is the diagnosis? 3

Balanitis circinata

Balanitis circinata presents as sharply demarcated, serpiginous ulcers or plaques on the penile head. Balanitis circinata is usually seen in Reiter's syndrome which has a classic triad of arthritis, urethritis, and conjunctivitis. The syndrome typically occurs post-infection of the GI or urinary tract. Q/Q(M)-476810 Report a Problem Almost all patients with SLE have positive ANAs. A patient can have ANA-negative SLE if they only make antibodies to: 1

ssDNA

2

Sm 27

3

U1RNP

4

Ro

5

dsDNA Q/Q(M)-480526 Report a Problem

Almost all patients with SLE have positive ANAs. A patient can have ANA-negative SLE if they only make antibodies to: 1

ssDNA

ANA-negative SLE is uncommon. The ANA has a high negative predictive value and low positive predictive value. ANA-negative SLE can result if a patient only makes antibodies to ssDNA, which is not detected by most tests. Q/Q(M)-480526 Report a Problem

A 62 year-old man with chronic renal failure on hemodialysis presents with carpal tunnel syndrome, bone cysts, and spondyloarthropathy. A diagnosis of amyloidosis is suspected. Which of the following is true regarding his most likely diagnosis? 1

AA amyloid is likely causative

2

AL amyloid is likely causative

3

beta 2-microglobulin is likely causative

4

keratin-derived amyloid is likely causative

5

Amyloid P component will not be found in affected tissues Q/Q(M)-480504 Report a Problem

A 62 year-old man with chronic renal failure on hemodialysis presents with carpal tunnel syndrome, bone cysts, and spondyloarthropathy. A diagnosis of amyloidosis is suspected. Which of the following is true regarding his most likely diagnosis? 3

beta 2-microglobulin is likely causative

The patient likely has dialysis-related amyloidosis. In this condition, beta 2-microglobulin is the protein component altered by uremia. Carpal tunnel syndrome, bone cysts, and spondyloarthropathy commonly result. Amyloid P component and ground substance are found in all forms of amyloidosis; the proteinderived amyloid fibers differ among the various forms. Q/Q(M)-480504 Report a Problem Which of the following is true regarding atopic dermatitis? 1

Over 80% of affected individuals present in the first year of life

2

Natural measles infection has been noted to improve atopic dermatitis

3

Most children develop worsening of atopic dermatitis if they develop asthma later in childhood 28

4

Staphylococcus aureus is found in ~25% of atopic dermatitis skin lesions

5

Increased expression of cathelicidins such as LL 37 has been noted in atopic dermatitis Q/Q(M)-480481 Report a Problem

Which of the following is true regarding atopic dermatitis? 2

Natural measles infection has been noted to improve atopic dermatitis

Natural measles infection has been noted to improve atopic dermatitis. Atopic dermatitis typically begins in infancy, with ~50% in the first year of life and an additional 30% between 1 and 5 years. Most children with AD eventually develop allergic rhinitis or asthma later in childhood. Many outgrow AD as respiratory allergy develops. S. aureus is found in over 90% of AD skin lesions. Decreased expression of innate antimicrobial peptides, such as human beta-defensin and cathelicidins, such as LL 37, may explain the increased susceptibility to colonization and skin infection with S. aureus in patients with atopic dermatitis. Q/Q(M)-480481 Report a Problem

Sarcoidosis presenting as uveitis, facial nerve palsy, fever and parotid gland swelling is referred to as: 1

Heerfordt's syndrome

2

Lofgren's syndrome

3

Lupus pernio

4

Darier-Roussy disease

5

Schaumann syndrome Q/Q(M)-480468 Report a Problem

Sarcoidosis presenting as uveitis, facial nerve palsy, fever and parotid gland swelling is referred to as: 1

Heerfordt's syndrome

Heerfordt's syndrome is the name given to sarcoidosis presenting with uveitis, facial nerve palsy, fever and parotid gland swelling. Lofgren's syndrome is an acute presentation of sarcoidosis that presents wth fever, arthritis, erythema nodosum and hilar adenopathy. Darier-Roussy disease is sarcoidosis presenting as painless firm subcutaneous nodules. Lupus pernio refers to sarcoidosis presenting as papulonodules and plaques involving the nose (especially the alar rim), ears and cheeks. Schaumann syndrome was a distractor as Shaumann bodies are seen in sarcoidosis on histopathology. Q/Q(M)-480468 Report a Problem

Which of the following is important in the pathogenesis of acne vulgaris? 1

Activation of toll-like receptor-3 by P. acnes

2

Activation of toll-like receptor-2 by M. furfur

3

Activation of toll-like receptor-2 by P. acnes

4

P. acnes produces lipase which cleaves cholesterol into triglycerides 29

5

Demodex activates complement Q/Q(M)-480419 Report a Problem

Which of the following is important in the pathogenesis of acne vulgaris? 3

Activation of toll-like receptor-2 by P. acnes

Acne vulgaris is a disease of follicular hyperkeratosis and the microcomedone is thought to be the precursor lesion. P. acnes has lipase that cleaves triglycerides into free fatty acids. P. acnes can activate complement and PMN chemotaxis. Toll-like receptors (TLR) recognize bacterial patterns and P. acnes activates TLR-2. Q/Q(M)-480419 Report a Problem

A child has a cutaneous ossification disorder characterized by endochondral bone formation, deafness, baldness, and mental retardation. Which form of osteoma cutis does this patient have? 1

Fibrodysplasia ossificans progressiva

2

Progressive osseous heteroplasia

3

Plate-like osteoma cutis

4

Albright's hereditary osteodystrophy

5

Calcinosis cutis Q/Q(M)-482391 Report a Problem

A child has a cutaneous ossification disorder characterized by endochondral bone formation, deafness, baldness, and mental retardation. Which form of osteoma cutis does this patient have? 1

Fibrodysplasia ossificans progressiva

This patient has fibrodysplasia ossificans progressiva characterized by endochondral bone formation (the other types have intramembranous bone formation), noggin gene defects and other systemic symptoms as listed. This is a progressive and potentially fatal condition. Progressive osseous heteroplasia is also progressive, seen in mostly females, and demonstrates increased alkaline phosphatase, LDH and CPK with normal calcium, phosphate, and PTH. Plate-like osteoma cutis is limited and seen in children and newborns. Albright's hereditary osteodystrophy is due to a mutation in GNAS-1. This disorder is characterized by a lack of responsiveness to parathyroid hormone, resulting in low serum calcium, high serum phosphate, and appropriately high serum parathyroid hormone. Individuals with Albright's hereditary osteodystrophy have short stature, characteristically shortened fourth and fifth metacarpals, rounded facies, and often mild mental retardation. Calcinosis cutis is not a form of osteoma cutis. Q/Q(M)-482391 Report a Problem

The mechanism of action of azithromycin is: 1

Inhibition of bacterial cell wall synthesis

2

Inhibition of RNA-dependent protein synthesis by binding to the 30s ribosomal subunit

3

Inhibition of RNA-dependent protein synthesis by binding to the 50s ribosomal subunit 30

4

Inhibition of DNA-dependent RNA polymerase

5

Inhibition of bacterial topoisomerase Q/Q(M)-474301 Report a Problem

The mechanism of action of azithromycin is: 3

Inhibition of RNA-dependent protein synthesis by binding to the 50s ribosomal subunit

Azithromycin is a macrolide antibiotic. It binds the bacterial 50s ribosomal subunit and inhibits RNAdependent protein synthesis. Q/Q(M)-474301 Report a Problem

A pregnant woman in her third trimester presents with nonpruritic erythematous plaques and pustules in the intertriginous regions, the trunk, and the extremities as well as systemic symptoms of malaise and fever. Which complication is most associated with this condition? 1

Premature labor

2

Placental insufficiency

3

No risk to fetus or mother

4

Small for gestational age

5

Microcephaly Q/Q(M)-482246 Report a Problem

A pregnant woman in her third trimester presents with nonpruritic erythematous plaques and pustules in the intertriginous regions, the trunk, and the extremities as well as systemic symptoms of malaise and fever. Which complication is most associated with this condition? 2

Placental insufficiency

This patient has pustular psoriasis of pregnancy which is also called "impetigo herpetiformis." It is characterized by red plaques with a peripheral ring of pustules that are distributed symmetrically in flexural areas and trunk and extremities. Patients can have elevated ESR and leukocytosis as well as hypocalcemia. There is a risk of fetal morbidity and mortality secondary to placental insufficiency and maternal mortality secondary to cardiac or renal failure. Premature labor may be associated with cholestasis or pregnancy. Patients with herpes gestationis may have a risk of premature labor and small for gestational age. Microcephaly may occur with isotretinoin taken during pregnancy. Q/Q(M)-482246 Report a Problem Acrokeratosis paraneoplastica is most commonly associated with which of the following malignancies: 1

Squamous cell carcinoma

2

Adenocarcinoma

3

Transitional cell carcinoma

4

T-cell lymphoma 31

5

Breast cancer Q/Q(M)-480556 Report a Problem

Acrokeratosis paraneoplastica is most commonly associated with which of the following malignancies: 1

Squamous cell carcinoma

Acrokeratosis paraneoplastica, also known as Bazex’s syndrome, is a rare paraneoplastic syndrome. Clinically, it appears as symmetric, hyperkeratotic lesions on red base, in an acral distribution, nose and helices of the ears. Most commonly associated with upper aerodigestive tract tumors. Treatment of the tumor leads to disappearance of lesions. Q/Q(M)-480556 Report a Problem

Which of the following is an example of a large vessel vasculitis? 1

Takayasu arteritis

2

Wegener's granulomatosis

3

Chrug-Strauss syndrome

4

Polyarteritis nodosa

5

Henoch-Schonlein purpura Q/Q(M)-480469 Report a Problem

Which of the following is an example of a large vessel vasculitis? 1

Takayasu arteritis

Takayasu arteritis is a large vessel vasculitis that manifests as progressive granulomatous inflammation of the aorta and its major branches. The systemic vasculitidies are classified into three categories: large vessel, medium-sized vessel and small vessel vasculitis. Takayasu arteritis and Giant cell (temporal) arteritis are the two large vessel vasculitidies. Polyarteritis nodosa and Kawasaki disease are mediumsized vasculitidies. Wegener's granulomatosis, Churg-Strauss syndrome, microscopic polyangiitis, Henoch-Schonlein purpura and cutaneous leukocytoclastic vasculitis are examples of small vessel vasculitidies. Q/Q(M)-480469 Report a Problem What mast cell mediators are preformed, rather than synthesized upon exposure to a trigger? 1

tryptase

2

leukotriene C4

3

thromboxane

4

prostaglandin D2

5

platelet activating factor Q/Q(M)-482861 Report a Problem

32

What mast cell mediators are preformed, rather than synthesized upon exposure to a trigger? 1

tryptase

Mast cell mediators can be grouped into two classes: preformed and newly formed. Preformed mediators include tryptase, histamine, serotonin, and heparin. Newly formed mediators include prostaglandin D2, leukotriene C4, and platelet activating factor. Q/Q(M)-482861 Report a Problem

What is the treatment of choice for this condition: 1

Topical antifungal medication

2

High potency topical steroid

3

Topical antibiotic cream

4

Oral antibiotic

5

Excision Q/Q(M)-476812 Report a Problem

What is the treatment of choice for this condition: 2

High potency topical steroid

The condition depicted is lichen sclerosus et atrophicus which most commonly affects the genital region. High potency topical steroids are the treatment of choice. Q/Q(M)-476812 Report a Problem

RNP antibodies include: 1

anti-SSA, anti-centromere, anti-SSB

2

anti-SSA, anti-SSB, anti-Sm, anti-U1RNP

3

anti-SSA, anti-SSB, anti-Sm, anti-dsDNA

4

ANA, anti-dsDNA

5

anti-SSA, anti-U1RNP, anti-centromere Q/Q(M)-480521 Report a Problem

33

RNP antibodies include: 2

anti-SSA, anti-SSB, anti-Sm, anti-U1RNP

RNP antibodies target small ribonucleoproteins. These include SS-A (Ro), SS-B (La), Sm, and U1RNP. The total amount of antibody has more diagnostic value than the mere presence of antibody. Q/Q(M)-480521 Report a Problem Which systemic anesthetic can be safely used in mastocytosis? 1

Lidocaine

2

Succinylcholine

3

D-tubocurarine

4

Thiopental

5

Propofol Q/Q(M)-482221 Report a Problem

Which systemic anesthetic can be safely used in mastocytosis? 5

Propofol

Local lidocaine can be safely used but systemic lidocaine can precipitate anaphylaxis in patients with mastocytosis. Other systemic analgesics that are counter-indicated in mastocytosis include succinylcholine, d-tubocurarine, thiopental, metocurine, enflurane, isoflurane, and etomidate. Propofol, vecuronium bromide, and fentanly are safe options for systemic anesthesia in patients with mastocytosis. Q/Q(M)-482221 Report a Problem

A 48 year old woman was recently diagnosed with dermatomyositis. Which examination would be most important in a work up for malignancy? 1

Breast exam

2

Chest xray

3

Thyroid exam

4

Pelvic exam

5

Stool guiac Q/Q(M)-477130 Report a Problem

A 48 year old woman was recently diagnosed with dermatomyositis. Which examination would be most important in a work up for malignancy? 4

Pelvic exam

Dermatomyositis is an idiopathic inflammatory disease with myositis and characteristic cutaneous manifestations. There is an increased incidence of malignancy in these patients which may precede, occur with or follow the diagnosis of dermatomyositis. The most common form of malignancy in adult women 34

is ovarian cancer. Other malignancies that have been associated include testicular cancer, gastrointestinal, lung and nasopharyngeal carcinomas. Q/Q(M)-477130 Report a Problem

A 17 year old high school cheerleader returns to clinic for follow up of her acne and management of her isotretinoin. She denies depression or any GI complications. She is pleased with her progress but does mention painful red bumps on her lower legs for 1 week. She attributes these to her cheer practice. What is the most likely cause of her lower extremity lesions. 1

Training regimen

2

Abusive boyfriend

3

Oral contraceptives

4

Allergy medication

5

Recent strep infection Q/Q(M)-482789 Report a Problem

A 17 year old high school cheerleader returns to clinic for follow up of her acne and management of her isotretinoin. She denies depression or any GI complications. She is pleased with her progress but does mention painful red bumps on her lower legs for 1 week. She attributes these to her cheer practice. What is the most likely cause of her lower extremity lesions. 3

Oral contraceptives

The patient is likely experiencing erythema nodosum (EN), a delayed hypersensitivity response to a wide variety of eliciting factors. They consist of an eruption of erythematous, tender nodules, typically over the anterior tibial areas. Common causes include oral contraceptives, which is a probable choice given that the patient is likely taking them as part of her isotretinoin treatment. Strep infection is also a common cause of EN but with no mention of symptoms is less likely in this case. Other causes include TB, fungal infections, sarcoidosis, ulcerative colitis, and regional enteritis. Q/Q(M)-482789 Report a Problem A 48 year-old man with a long history of alcohol abuse and dependence presents with angular cheilitis, atrophic glossitis, a scrotal dermatitis that spares the midline and extends to the thighs, photophobia, and blepharitis. Which nutritional deficiency do you suspect? 1

Vitamin B12

2

Vitamin C

3

Pyridoxine

4

Vitamin B2

5

Iron Q/Q(M)-480554 Report a Problem

35

A 48 year-old man with a long history of alcohol abuse and dependence presents with angular cheilitis, atrophic glossitis, a scrotal dermatitis that spares the midline and extends to the thighs, photophobia, and blepharitis. Which nutritional deficiency do you suspect? 4

Vitamin B2

The patient presents with classic findings of the oral-ocular-genital syndrome seen in the setting of vitamin B2 (riboflavin) deficiency. This deficiency occurs most often in alcoholics. It characteristically presents with angular cheilitis, atrophic glossitis (magenta), a seborrheic-like dermatitis around the nose, genital dermatitis (scrotal dermatitis sparing the midline and extending to the thighs), photophobia, and blepharitis. Q/Q(M)-480554 Report a Problem

Which of the following medications is NOT associated with exacerbating or causing this eruption? 1

Lithium

2

Prednisone

3

Phenytoin

4

Isoniazid

5

Propranolol Q/Q(M)-476826 Report a Problem

Which of the following medications is NOT associated with exacerbating or causing this eruption? 5

Propranolol

Acne and acneiform eruptions can be caused or exacerbated by cortiosteroids, oral contraceptives, androgens, ACTH, lithium, phenytoin, halogens, INH, and haloperidol. Q/Q(M)-476826 Report a Problem

Which of the following is true regarding this entity? 1

The subcutaneous variant is most common in children

2

75% of localized lesions are still present 2 years after diagnosis

3

Localized lesions often ulcerate and heal with scarring

4

The perforating variant classically involves the lower extremities

5

Diabetes is present in the majority of patients with the generalized variant 36

Q/Q(M)-480492 Report a Problem

Which of the following is true regarding this entity? 1

The subcutaneous variant is most common in children

Granuloma annulare is the entity pictured. Subtypes include localized GA, generalized GA, macular GA, subcutaneous GA, and perforating GA. The subcutaneous variant is most common in children; there is often a history of trauma to the area. Localized lesions never ulcerate and heal without scarring; 75% of localized lesions clear within 2 years. The perforating variant classically involves the dorsa of the hands. Diabetes is present in 20% of patients with the generalized variant. Q/Q(M)-480492 Report a Problem

A 35 year-old woman presents with predominantly lower facial inflammatory acne, hirsutism, and irregular menses. She has failed multiple conventional treatments. Laboratory work-up reveals a very high level of DHEA-S. You are most concerned about: 1

Congenital adrenal hyperplasia

2

Adrenal tumor

3

Polycystic ovary syndrome

4

Toxicity from prolonged use of spironolactone

5

Progestin excess from oral contraceptives Q/Q(M)-480381 Report a Problem

A 35 year-old woman presents with predominantly lower facial inflammatory acne, hirsutism, and irregular menses. She has failed multiple conventional treatments. Laboratory work-up reveals a very high level of DHEA-S. You are most concerned about: 2

Adrenal tumor

A patient whose acne fails to respond to conventional therapy, whose acne flares cyclically, with hirsutism, alopecia, or irregular menses warrants an endocrine work-up, including free and total testosterone, LH, FSH, and DHEA-S. Although rare, very high levels of DHEA-S may suggest an adrenal adrogen-secreting tumor. Q/Q(M)-480381 Report a Problem

37

The dermatosis pictured is most commonly associated with: 1

Insulin resistance

2

Cushing's syndrome

3

Acromegaly

4

Polycystic ovarian syndrome

5

Gastric carcinoma Q/Q(M)-480479 Report a Problem

The dermatosis pictured is most commonly associated with: 1

Insulin resistance

Insulin resistance is the most common cause of acanthosis nigricans. Insulin-like growth factors, produced by the liver in response to high levels of circulating insulin, bind epidermal growth factor receptors to produce thickening of the epidermis and hyperkeratosis. Cushing's syndrome, acromegaly and polycystic ovarian syndrome are associated with acanthosis nigricans, although less common than insulin resistance. Paraneoplastic acanthosis nigricans is associated with gastrointestinal carcinoma, classically gastric carcinoma. Q/Q(M)-480479 Report a Problem

Eyebrow growth: 1

Is androgen-dependent in men but not in women

2

Is regulated by dehydroepiandrosterone but not androstenedione

3

Is not androgen-dependent

4

Requires conversion of testosterone to dihydrotestosterone in the hair follicle

5

Is regulated only by testosterone and dihydrotestosterone Q/Q(M)-480498 Report a Problem

Eyebrow growth: 38

3

Is not androgen-dependent

Eyebrows, eyelashes, and vellus hair are not androgen-dependent, thus there is no difference between these areas of hair growth in men and women. Q/Q(M)-480498 Report a Problem

Leukokeratosis nicotina palati: 1

Is not attributed to smoking

2

Represents a premalignant condition

3

Responds to intralesional steroids

4

Represents inflamed minor salivary glands

5

Resolves with tetracycline therapy Q/Q(M)-477172 Report a Problem

Leukokeratosis nicotina palati: 4

Represents inflamed minor salivary glands

Leukokeratosis nicotina palati, also called smoker�s palate, is a complication of tobacco use, especially pipe smokers. Lesions develop as a result of inflamed minor salivary glands and manifest as uniform keratosis of the hard palate with multiple red, umbilicated papules. Q/Q(M)-477172 Report a Problem

Which of the following statements comparing inflammatory lesions from patients with psoriasis and patients with atopic dermatitis is true? Human �-defensin 2 (HBD-2) and LL 37 levels in psoriasis lesions are significantly lower than in 1 atopic dermatitis lesions 2

Expression of HBD-2 and LL 37 is negligible in psoriatic lesions

3

HBD-2 and LL 37 are significantly elevated in atopic dermatitis lesions

4 5

Expression of HBD-2 and LL 37 in psoriasis is similar to normal skin, whereas it is decreased in atopic dermatitis lesions HBD-2 and LL 37 are significantly decreased in atopic dermatitis lesions compared to psoriasis lesions Q/Q(M)-474308 Report a Problem

Which of the following statements comparing inflammatory lesions from patients with psoriasis and patients with atopic dermatitis is true? HBD-2 and LL 37 are significantly decreased in atopic dermatitis lesions compared to psoriasis 5 lesions Ong et al found a deficiency of HBD-2 and LL 37, which are innate antimicrobial peptides, in lesions from patients with atopic dermatitis compared to lesions from patients with psoriasis. IL-4 and IL-13, which are Th2-type cytokines, were found to inhibit expression of HBD-2 and LL 37. Decreased expression of HBD-2 and LL 37 may account for the increased susceptibility to S. aureus infection in 39

patients with atopic dermatitis. Q/Q(M)-474308 Report a Problem Pernio or Chilblains is an inflammatory skin condition which is triggered by cold, wet, non-freezing environmental conditions. Acral skin has violaceous discoloration accompanied by burning or itching. While avoidance and prevention is best, the most effective pharmacologic treatment is: 1

Nifedipine

2

Nicotinamide

3

Phenoxybenzamine

4

Psoralen+UVA

5

Aspirin Q/Q(M)-477646 Report a Problem

Pernio or Chilblains is an inflammatory skin condition which is triggered by cold, wet, non-freezing environmental conditions. Acral skin has violaceous discoloration accompanied by burning or itching. While avoidance and prevention is best, the most effective pharmacologic treatment is: 1

Nifedipine

Nifedipine is effective in about 70% of patients with pernio in prevention of the development of new skin lesions. The other options listed, other than aspirin, are anecdotally suggested to be helpful. Q/Q(M)-477646 Report a Problem A patient develops pulmonary fibrosis, Raynaud's phenomenon and skin changes similar to systemic sclerosis. These changes resolve following discontinuation of which of the following medications? 1

Bleomycin

2

Penicillamine

3

Hydralazine

4

Minocycline

5

Isoniazide Q/Q(M)-480066 Report a Problem

A patient develops pulmonary fibrosis, Raynaud's phenomenon and skin changes similar to systemic sclerosis. These changes resolve following discontinuation of which of the following medications? 1

Bleomycin

The changes described above are similar to findings of scleroderma. Patients receiving bleomycin can develop a reversible syndrome similar to scleroderma. The other four listed options are related to druginduced lupus erythematosus and are not linked to scleroderma-like syndromes. Q/Q(M)-480066 Report a Problem

40

What is the most likely diagnosis? 1

Voight line

2

Linea nigra

3

Fuchter line

4

Phytophotodermatitis

5

Incontinentia pigmenti Q/Q(M)-476887 Report a Problem

What is the most likely diagnosis? 2

Linea nigra

Linea nigra is a hyperpigmented, linear patch that often becomes apparent during pregnancy. It is thought to be secondary to hormonal fluctuations and usually spontaneously resolves after pregnancy. Voight and Fuchter lines delineate a vestigial hyperpigmentation of the dorsal aspect of organisms. Q/Q(M)-476887 Report a Problem

Which of the following viruses is associated the most with lichen planus? 1

Hepatitis C virus

2

Hepatitis B virus

3

Human immunodeficiency virus

4

Hepatitis A virus

5

Epstein-barr virus Q/Q(M)-482811 Report a Problem

Which of the following viruses is associated the most with lichen planus? 1

Hepatitis C virus

Lichen planus classically involves the wrists, ankles, and oral mucosa. It is characterized by violaceous polygonal, purple, pruritic papules. It has a strong association with hepatitis C virus. Many other associations exist, including other viruses and medications, but hepatitis C is the best choice. 41

Q/Q(M)-482811 Report a Problem

Which of the following is NOT true regarding primary cutaneous amyloidosis? 1

AA is not the protein component

2

Amyloid is present around blood vessels

3

AL is not the protein component

4

Notalgia paresthetica may be associated with the macular form

5

Amyloid found in benign and malignant neoplasms does not represent primary cutaneous amyloidosis Q/Q(M)-480505 Report a Problem

Which of the following is NOT true regarding primary cutaneous amyloidosis? 1

AA is not the protein component

2

Amyloid is present around blood vessels

3

AL is not the protein component

4

Notalgia paresthetica may be associated with the macular form

5

Amyloid found in benign and malignant neoplasms does not represent primary cutaneous amyloidosis Q/Q(M)-480505 Report a Problem

Which of the following is NOT true regarding primary cutaneous amyloidosis? 2

Amyloid is present around blood vessels

Keratin is the protein component in primary cutaneous amyloidosis. No amyloid is found around blood vessels. Macular amyloidosis (which may have associated notalgia paresthetica) and lichen amyloidosis are forms of primary cutaneous amyloidosis. Secondary cutaneous amyloidosis presents with keratinderived amyloid and is seen following PUVA therapy and in benign and malignant neoplasms. Q/Q(M)-480505 Report a Problem

The best diagnosis for this congenital melanopenic lesion without extracutaneous associations is 1

Hypomelanosis of Ito

2

Segmental vitiligo

3

Ash leaf spots

4

Nevus anemicus

5

Nevus achromicus Q/Q(M)-482125 Report a Problem 42

The best diagnosis for this congenital melanopenic lesion without extracutaneous associations is 5

Nevus achromicus

Nevus achromicus is another name for nevus depigmentosus. It usually presents at birth or appear during early infancy as normal pigmentation increases. Most individuals will have a solitary lesion of nevus depigmentosus, but multiple lesions and segmental forms of nevus depigmentosus have been described. Nevus depigmentosus tends to persist lifelong, but remains unchanged after onset. The hypopigmented white spots of tuberous sclerosis are most difficult to distinguish from nevus depigmentosus, but lack of other cutaneous or systemic manifestaions exclude tubrous sclerosis. Lesions of vitiligo tend to be depigmented (melanocytopenic not melanopenic as in the question), and show a bright white coloration with Wood's lamp examination. Nevus anemicus is a distinct vascular birthmark characterized by blanching of cutaneous blood vessels, hence presenting as a "white" patch of skin that becomes unnoticeable when the surrounding skin is blanched with a glass slide ("diascopy"). Q/Q(M)-482125 Report a Problem

Scalp biopsy of a 44-year-old female with suspected discoid lupus erythematosis would likely reveal inflammation around which portion of the hair follicle: 1

Dermal papilla

2

Hair bulb

3

Matrix

4

Isthmus

5

Infundibulum Q/Q(M)-482937 Report a Problem

Scalp biopsy of a 44-year-old female with suspected discoid lupus erythematosis would likely reveal inflammation around which portion of the hair follicle: 4

Isthmus

On biopsy, alopecia areata exhibits a peribulbar lymphocytic �swarm of bees.� Discoid lupus typically exhibits inflammation surrounding the isthmus, along with a perivascular dermatitis and vacuolar interface changes. Lichen planopilaris exhibits inflammation most densely concentrated about the infundibulum. Q/Q(M)-482937 Report a Problem

43

A 22 year-old woman notes irregular menses and acne that has been refractory to several over-thecounter and prescription regimens. Appropriate initial work-up includes 1

Serum prolactin levels

2

24-hour urine cortisol

3

Serum free and total testosterone, LH, FSH, androstenedione

4

Serum free and total testosterone, LH, FSH, DHEA-S

5

Serum free and total testosterone, LH, FSH, 17-OH, hydroxylase Q/Q(M)-474890 Report a Problem

A 22 year-old woman notes irregular menses and acne that has been refractory to several over-thecounter and prescription regimens. Appropriate initial work-up includes 4

Serum free and total testosterone, LH, FSH, DHEA-S

The history and clinical appearance of this patient suggest a possible hormonal basis to her acne. The work-up in answer d assesses for adrenal hormonal production and screens for PCOS. Androstenedione is of ovarian origin. Q/Q(M)-474890 Report a Problem This radiograph shows a characteristic finding of severe psoriatic arthritis: bone proliferation at the base of the distal phalanx with resorption of the tufts. What is the most common form of psoriatic arthritis? 1

Arthritis mutilans

2

Axial

3

Symmetric polyarthritis

4

Asymmetric oligoarthritis

5

Symmetric oligoarthritis Q/Q(M)-474895 Report a Problem

This radiograph shows a characteristic finding of severe psoriatic arthritis: bone proliferation at the base of the distal phalanx with resorption of the tufts. What is the most common form of psoriatic arthritis? 4

Asymmetric oligoarthritis 44

Approximately 70 % of psoriatic arthritis is asymmetric oligoarthritis. The slide depicts arthritis mutilans, which occurs in about 5% of patients with psoriatic arthritis. Q/Q(M)-474895 Report a Problem

Which HLA type is more commonly associated with this clinical finding? 1

HLA-B7

2

HLA-B15

3

HLA-B27

4

HLA-B51

5

HLA-DR4 Q/Q(M)-476811 Report a Problem

Which HLA type is more commonly associated with this clinical finding? 3

HLA-B27

The condition shown is balnitis circinata which is part of the constellation of findings in Reiter's syndrome in addition to arthritis, urethritis, and conjunctivitis. There is a higher incidence of this condition people with HLA-B27. Q/Q(M)-476811 Report a Problem

This patient was started on isotretinoin but failed to discontinue the tetracycline. He is at risk for what complication? 1

Acne fulminans

2

Pseudotumor cerebri

3

Diarrhea

4

Depression

5

Myositis Q/Q(M)-476889 Report a Problem

45

This patient was started on isotretinoin but failed to discontinue the tetracycline. He is at risk for what complication? 2

Pseudotumor cerebri

Both isotretinoin and tetracycline are known to cause pseudotumor cerebri. In combination, the risk is significantly elevated. Symptoms of pseudotumor cerebri include headaches, pulsatile tinnitus, diplopia, and blurred vision. Q/Q(M)-476889 Report a Problem

Bullous lupus erythematosus is most commonly associated with antibodies to: 1

Type IV collagen

2

Laminin 5

3

Desmoglein I

4

Plectin

5

Type VII collagen Q/Q(M)-482145 Report a Problem

Bullous lupus erythematosus is most commonly associated with antibodies to: 5

Type VII collagen

Type VII collagen is found in the sublamina densa and patients with bullous lupus erythematosus have been found to have antibodies to this protein. Bullous LE and EBA share antibasement membrane zone antibodies of identical specificity (type VII collagen) as well as clinical and histologic overlap. A differentiating feature between bullous LE and EBA is that bullous LE has a dramatic response to dapsone. Q/Q(M)-482145 Report a Problem

What is the most common infection that can cause guttate psoriasis? 1

Streptococcus pyogenes

2

Coccidiomycosis

3

Herpes

4

Mycoplasma pneumonia 46

5

Tuberculosis Q/Q(M)-482069 Report a Problem

What is the most common infection that can cause guttate psoriasis? 1

Streptococcus pyogenes

Guttate psoriasis refers to a distinctive, acute clinical presentation of an eruption characterized by small, droplike, 1-10 mm in diameter, salmon-pink papules, usually with a fine scale. It is more commonly seen in individuals younger than 30 years with a history of upper respiratory infection that precedes the eruption by 2-3 weeks. The most common organism is group A beta-hemolytic streptococci (eg, Streptococcus pyogenes). Although recurrent episodes may occur, especially those due to pharyngeal carriage of streptococci, isolated bouts are known to occur. Guttae psoriasis may also occur with other infections or in isolation without any identifiable infection. Q/Q(M)-482069 Report a Problem

Which class of medication works by inhibits DNA gyrase? 1

Quinolones

2

Tetracyclines

3

Aminoglycosides

4

Cephalosporins

5

Carbapenems Q/Q(M)-482797 Report a Problem

Which class of medication works by inhibits DNA gyrase? 1

Quinolones

Penicillins, cephalosporins, carbapenems, and vancomycin work on the cell wall. Quinolones inhibit DNA gyrase. Tetracyclines and aminoglycosides work at the 30 S ribosomal subunit. Q/Q(M)-482797 Report a Problem Which of the following disorders is most strongly associated with a monoclonal gammopathy of the IgGlambda type? 1

Scleredema 47

2

POEMS syndrome

3

Necrobiotic xanthogranuloma

4

EED

5

Scleromyxedema Q/Q(M)-482477 Report a Problem

Which of the following disorders is most strongly associated with a monoclonal gammopathy of the IgGlambda type? 5

Scleromyxedema

All of the above conditions are assocaited with some type of monoclonal gammopathy. Scleromyxedema is associated with an IgG-lambda monoclonal gammopathy. Scleredema and NXG are most often associated with IgG-kappa, and both POEMS syndrome and EED are usually associated with IgA monoclonal gammopathies. Q/Q(M)-482477 Report a Problem A pregnant woman presents with mild inflammatory acne which is very bothersome to her. Which of the following is the most appropriate treatment option based on FDA classifications of medication in pregnancy? 1

Topical erythromycin/benzoyl peroxide gel

2

Topical tretinoin

3

Tazarotene 0.1% cream

4

Azelaic acid 20% cream

5

Bactrim Q/Q(M)-474341 Report a Problem

A pregnant woman presents with mild inflammatory acne which is very bothersome to her. Which of the following is the most appropriate treatment option based on FDA classifications of medication in pregnancy? 4

Azelaic acid 20% cream

Azeleic acid is category B in pregnancy. The others are category C (benzoyl peroxide, topical tretinoin, and bactrim) or category X (tazarotene). The categories for safety of drugs in pregnancies are as follows: Category A: controlled studies in humans show no risk to fetus. Category B: controlled human studies show no risk to fetus but may show risk to animals, or no risk in animal studies but no human studies conducted. Category C: risk to human fetus cannot be ruled out, studies are lacking; animal studies equivocal. Category D: controlled suties show risk to human fetus, benefits may sometimes outweigh risk. Cateogory X: contraindicated in pregnancy. Q/Q(M)-474341 Report a Problem An 8 year old healthy boy complaining of changes on his nails for the past 10 months. What is the most likely diagnosis? 48

1

Psoriasis

2

Alopecia

3

Pachynonychia congenita

4

Chronic mucocutaneous candidiasis

5

Atopic dermatitis Q/Q(M)-482880 Report a Problem

An 8 year old healthy boy complaining of changes on his nails for the past 10 months. What is the most likely diagnosis? 1

Psoriasis

Nail involvement in psoriasis is common. It affects about 25-50% of pediatric patients with psoriasis, more commonly during the second decade of life. Q/Q(M)-482880 Report a Problem

Spindle cell lipoma is commonly found on the: 1

Head

2

Lower extremities

3

Buttocks

4

Breast

5

Posterior shoulder Q/Q(M)-477681 Report a Problem

Spindle cell lipoma is commonly found on the: 5

Posterior shoulder

Spindle cell lipoma is a solitary benign tumor seen in adult male patients, and is most often located on the posterior shoulder and neck regions.The tumor histologically consists of mature collagen, adpose tissue, spindle cells, and mast cells. Treatment is with local excision. Q/Q(M)-477681 Report a Problem Which of the following forms of angioedema is mediated by kinin release? 1

Hereditary angioedema

2

Acute allergic angioedema

3

Angioedema associated with urticarial vasculitis

4

NSAID angioedema

5

Infectious angioedema 49

Q/Q(M)-482506 Report a Problem

Which of the following forms of angioedema is mediated by kinin release? 1

Hereditary angioedema

Types of angioedema mediated through kinin release include hereditary angioedema, acquired C1 inhibitor deficiency, and ACE inhibitor associated angioedema. These conditions are not associated with concurrent urticaria. Q/Q(M)-482506 Report a Problem

What is the most common site of extracutaneous involvement in this non-infectious disease? 1

Lungs

2

Eyes

3

Bone Marrow

4

Liver

5

Nasal mucosa Q/Q(M)-476631 Report a Problem

What is the most common site of extracutaneous involvement in this non-infectious disease? 1

Lungs

Sarcoidosis is a non-infectious granulomatous disorder of unclear etiology. It has a predilection for the lungs (90%), lymph nodes (75-90%), eyes (25%), nasal mucosa (20%), bone marrow (25-40%), and liver (15-40%). Q/Q(M)-476631 Report a Problem

A positive ANA with a speckled staining pattern correlates with staining of what cellular component? 1

Ribonucleoproteins

2

Nucleolar RNA

3

Native DNA

4

Kinetochore

5

Histones Q/Q(M)-480464 Report a Problem 50

A positive ANA with a speckled staining pattern correlates with staining of what cellular component? 1

Ribonucleoproteins

A speckled ANA pattern correlates with staining of ribonucleoproteins and is seen in Mixed Connective Tissue Disease (MCTD), Systemic Lupus Erythematosus (SLE), systemic sclerosis and Sjogren's syndrome. A nucleolar staining pattern correlates with staining of nucleolar RNA. Centromere staining correlates with staining of kinetochore. A peripheral staining pattern correlates with staining of native DNA and a homogeneous staining pattern correlates with staining of native DNA and histones. Q/Q(M)-480464 Report a Problem

A common site for chloracne is the: 1

Occipital scalp

2

Forehead

3

Scrotum

4

Forearms

5

Shoulders Q/Q(M)-474330 Report a Problem

A common site for chloracne is the: 3

Scrotum

Common sites for chloracne include the malar cheek, the post-auricular scalp and in men, the scrotum. Q/Q(M)-474330 Report a Problem

An 8 year old boy with seasonal allergies presents with red brown macules on the trunk present for several months. Parent mentions that when these lesions are scratched they form an irritated, red wheal. Which of the following sign is likely described? 1

Asboe-Hansen sign

2

Homan’s sign

3

Darier’s sign

4

Auspitz sign

5

Hutchinson sign Q/Q(M)-482790 Report a Problem

An 8 year old boy with seasonal allergies presents with red brown macules on the trunk present for several months. Parent mentions that when these lesions are scratched they form an irritated, red wheal. Which of the following sign is likely described? 51

Darier’s sign

3

Darier’s sign is seen in patients with urticarial pigmentosa, a common form of mastocytosis. Erythema and wheals are commonly elicited with stroking or rubbing secondary to mast cell degranulation. Symptoms can range from very mild (flushing, hives, no treatment needed) to life-threatening (vascular collapse). Asboe Hansen, commonly seen in pemphigus vulgaris, refers to extension of a bulla to adjacent unblistered skin when pressure is put on top of a bulla. Q/Q(M)-482790 Report a Problem

The diagnosis is: 1

Onychomycosis

2

Trauma-induced nail changes

3

Chronic paronychia

4

Pseudomonal infection

5

Psoriasis arthritis Q/Q(M)-474891 Report a Problem

The diagnosis is: 5

Psoriasis arthritis

This slide shows lateral onycholysis and subtle swelling of the distal interphalangeal joint (DIP), both characteristic findings of psoriasis and psoriatic arthritis. Q/Q(M)-474891 Report a Problem

You suspect that a patient has acquired angioedema. Levels of which of the following would you expect to be low? 1

C1q, C3

2

C1q, bradykinin

3

C4, C1q

4

bradykinin, C3 52

5

C4, C3 Q/Q(M)-482452 Report a Problem

You suspect that a patient has acquired angioedema. Levels of which of the following would you expect to be low? 3

C4, C1q

Initial screening test in angioedema for both inherited and acquired should include C4, which is low. C3 is normal in angioedema. C1q is low in acquired angioedema but normal in the hereditary type. Bradykinin is elevated in both inherited and acquired angioedema. Q/Q(M)-482452 Report a Problem

A 40 year-old man presents with a complaint of nail changes for several years. He was treated by an outside physician with terbinafine without improvement. On further questioning, he reports morning shoulder stiffness and back pain that lasts 1-2 hours and improves with activity. Which of the following is true regarding his condition? 1

50% of patients present with joint disease prior to skin involvement

2

Bony erosions are not commonly seen on radiographs

3

Cyclosporine is effective in treating the arthritis in this condition

4

A positive rheumatoid factor may be seen

5

Joint disease correlates with severity of skin involvement Q/Q(M)-480388 Report a Problem

A 40 year-old man presents with a complaint of nail changes for several years. He was treated by an outside physician with terbinafine without improvement. On further questioning, he reports morning shoulder stiffness and back pain that lasts 1-2 hours and improves with activity. Which of the following is true regarding his condition? 4

A positive rheumatoid factor may be seen

This patient has psoriatic nail changes and a history suggestive of psoriatic arthritis. Psoriatic arthritis is an inflammatory arthropathy associated with psoriasis. In 80% of patients the rheumatoid factor is negative; however a positive rheumatoid factor may sometimes be seen. 80% of patients present with 53

skin disease first. Large eccentric erosions are classically present on radiographs. Cyclosporine is not effective in treating psoriatic arthritis. Mild skin disease may be associated with moderate-to-severe joint disease, and vice versa. Q/Q(M)-480388 Report a Problem

Which form of sarcoidosis is associated with camptodactyly? 1

Lofgren's syndrome

2

Heerfordt's syndrome

3

Darier-Roussy

4

Mikulicz syndrome

5

Blau syndrome Q/Q(M)-477494 Report a Problem

Which form of sarcoidosis is associated with camptodactyly? 5

Blau syndrome

Camptodactyly is a flexion contracture of the 3rd through the 5th proximal interphalangeal joints and elbows bilaterally. Blau syndrome is an autosomal dominant form of sarcodosis due to a defect in the CARD15 gene manifesting also with arthritis, cutaneous sarcoid, uveitis, and synovial cysts. It does not have lung or visceral involvement. Q/Q(M)-477494 Report a Problem A patient complained of sudden appearance of multiple papules on chest and upper arms. Preliminary report by the dermpath fellow are cystic spaces lines by two layers of cuboidal cells and epithelial strands of similar cells. What is the most likely diagnosis? 1

Syringomas

2

Hydrocystomas

3

Acrospiromas

4

Spiradenomas

5

Mixed tumors Q/Q(M)-482885 Report a Problem

A patient complained of sudden appearance of multiple papules on chest and upper arms. Preliminary report by the dermpath fellow are cystic spaces lines by two layers of cuboidal cells and epithelial strands of similar cells. What is the most likely diagnosis? 1

Syringomas

The eruptive type of syringomas. In addition to the description above, some of the cysts have small comma-like tails, which is known as a �tadpole� pattern. Eruptive syringomas are histologically 54

identical to those on the eyelid, but appear suddenly on the neck, chest, axillae, upper arms, and periumbilically. It usually occurs in young people. The other options do not have an eruptive form and tend to occur in different locations of the body other than the arms and thorax. Q/Q(M)-482885 Report a Problem What is the most common paraproteinemia in scleromyxedema 1

IgA

2

IgG kappa

3

IgG lambda

4

IgM

5

It is uncommon to see paraproteinemia with scleromyxedema Q/Q(M)-482148 Report a Problem

What is the most common paraproteinemia in scleromyxedema 3

IgG lambda

An abnormal paraproteinemia is found in approximately 90% of cases with scleromyxedema, generally IgG lambda. IgA paraproteinemia can be seen in pyoderma gangrenosum and Sneddon-Wilkinson disease. Amyloidosis and NXG can be associated with IgG kappa paraproteinemia, and Schnitzler syndrome is associated with an IgM paraproteinemia. Q/Q(M)-482148 Report a Problem What mast cell mediators are synthesized upon exposure to a trigger, rather than preformed? 1

tryptase

2

histamine

3

serotonin

4

prostaglandin D2

5

heparin Q/Q(M)-482849 Report a Problem

What mast cell mediators are synthesized upon exposure to a trigger, rather than preformed? 4

prostaglandin D2

Mast cell mediators can be grouped into two classes: preformed and newly formed. Preformed mediators include tryptase, histamine, serotonin, and heparin. Newly formed mediators include prostaglandin D2, leukotriene C4, and platelet activating factor. Q/Q(M)-482849 Report a Problem

Which of the following is not a common food or environmental allergen implicated in atopic dermatitis? 55

1

Dermatophagoides pteronyssimus

2

Wheat

3

Corn

4

Eggs

5

Fish Q/Q(M)-480483 Report a Problem

Which of the following is not a common food or environmental allergen implicated in atopic dermatitis? 3

Corn

While most AD patients do not have food allergy, food allergens exacerbate AD in at least a subset of patients, particularly infants and young children. Eggs, milk, peanuts, soybeans, tree nuts, fish, and wheat are the most common food allergens implicated. Dust mites (Dermatophagoides pteronyssimus) are among the environmental allergens that may exacerbate AD. Q/Q(M)-480483 Report a Problem Necrobiotic xanthogranuloma is most commonly associated with which of the following: 1

IgA monoclonal gammopathy

2

IgM monoclonal gammopathy

3

IgG lambda monoclonal gammopathy

4

IgG kappa monoclonal gammopathy

5

IgG/IgA polyclonal gammopathy Q/Q(M)-482684 Report a Problem

Necrobiotic xanthogranuloma is most commonly associated with which of the following: 4

IgG kappa monoclonal gammopathy

Necrobiotic xanthogranuloma is associated with a monoclonal IgG kappa paraproteinemia in approximately 80% of cases. It is less commonly associated with an IgA gammopathy. Myeloma or myelodysplastic syndromes may resultantly occur. Q/Q(M)-482684 Report a Problem The main cause of nutritional disease in developed nations is: 1

Unusual diets

2

Inflammatory bowel disease

3

Malabsorption syndromes

4

Alcoholism

5

Psychiatric illness 56

Q/Q(M)-480552 Report a Problem

The main cause of nutritional disease in developed nations is: 4

Alcoholism

Alcoholism is the main cause of nutritional disease in developed nations. Other conditions that cause nutritional disease include: unusual diets, postoperative state, psychiatric illness, inflammatory bowel disease, cystic fibrosis, surgical bowel dysfunction, and inborn errors of metabolism. Q/Q(M)-480552 Report a Problem A 45-year-old woman develops bilateral poikilodermatous patches without atrophy on her lateral thighs. This is most likely a manifestation of what disease? 1

Lupus Erythematosus

2

Dermatomyositis

3

Mycosis Fungoides

4

Hepatitis C

5

Diabetes Mellitus Q/Q(M)-482218 Report a Problem

A 45-year-old woman develops bilateral poikilodermatous patches without atrophy on her lateral thighs. This is most likely a manifestation of what disease? 2

Dermatomyositis

Poikilodermatous changes on the lateral thigh represent the "holster sign"€ • seen in dermatomyositis. The poikilodermatous patches seen in dermatomyositis are often symmetric macular violaceous erythemas with dyspigmentation. Other characteristic poikilodermatous manifestations of dermatomyositis include the "V-sign" on the anterior neck and upper chest and the "shawl sign"€™ on the posterior neck, upper back, and the posterior shoulders. Lesions of Mycosis Fungoides typically have some component of epidermal atrophy. Q/Q(M)-482218 Report a Problem Which of the following is characteristic of lichen planus pigmentosus? 1

Lesions are typically hypopigmented

2

Most cases present in Caucasians

3

Oral involvement is pathognomonic

4

Can occasionally see epidermotropic T-cells

5

Trunk is typically spared Q/Q(M)-482521 Report a Problem

Which of the following is characteristic of lichen planus pigmentosus? 4

Can occasionally see epidermotropic T-cells 57

Occasionally epidermotropic T-cells are seen in the lichenoid reaction and thus may raise concern for mycosis fungoides. Lichen planus pigmentosus inversus, in particular, presents in classic sites of mycosis fungoides, including the axilla, inguinal, and inframammary areas. The individual lesions of lichen planus pigmentosis are typically smaller, however, than those encountered in mycosis fungoides, thus helping with the differential. Lesions of lichen planus pigmentosus are hyperpigmented, with most cases prsenting in skin of color. Oral involvement is rare and the trunk can be involved. Q/Q(M)-482521 Report a Problem

A concerned 23 year old male presents to clinic with multiple verrucoid lesions on his penis. He is anxious to have them removed. You explain that these are commonly treated cryosurgically. What is the boiling point of liquid nitrogen in Celsius? 1

-89.5

2

-40.8

3

3.8

4

-195.8

5

-78.5 Q/Q(M)-482791 Report a Problem

A concerned 23 year old male presents to clinic with multiple verrucoid lesions on his penis. He is anxious to have them removed. You explain that these are commonly treated cryosurgically. What is the boiling point of liquid nitrogen in Celsius? 4

-195.8

The boiling point of liquid nitrogen is -195.8. Nitrous oxide is -89.5. Carbon dioxide is -78.5. Dichlorotrifluoroethane (Freon 114) is 3.8. Chlorodifluoromethane (Freon 22) is 40.8. Colder cryogen is needed for adequate deep destruction, making liquid nitrogen the only reliable agent of the five listed. The Freons, solid CO2 and nitrous oxide are used only for topical anesthesia and superficial destruction. Q/Q(M)-482791 Report a Problem

Sneddon-Wilkinson Disease: 1

Most often occurs in elderly women

2

Rarely involves intertriginous areas

3

May occur in association with an IgG monoclonal gammopathy

4

Can be treated with narrow band UVB

5

Is usually an acute, self-limited condition Q/Q(M)-480395 Report a Problem

Sneddon-Wilkinson Disease: 4

Can be treated with narrow band UVB 58

Sneddon-Wilkinson disease, or subcorneal pustular dermatosis, presents with superficial pustules in annular and serpiginous patterns in the axillae, groin, and abdomen. Middle-aged women are most often affected. This condition rarely occurs in association with an IgA monoclonal gammopathy. It is a chronic condition, possibly related to psoriasis, with remissions of variable duration. Treatments including dapsone, acitretin, and narrow band UVB. Q/Q(M)-480395 Report a Problem Which of the following HLA types is associated with early onset psoriasis? 1

HLA-B13 or HLA-B17

2

HLA-B27

3

HLA-B57, HLA-Cw6, or HLA-DR7

4

HLA-Cw2

5

HLA-DR3 Q/Q(M)-482152 Report a Problem

Which of the following HLA types is associated with early onset psoriasis? 3

HLA-B57, HLA-Cw6, or HLA-DR7

HLA-B57, HLA-Cw6, or HLA-DR7 are most commonly associated with early onset type I psoriasis. The presence of HLA-B13 or B17 is associated with a 5-fold risk of developing psoriasis and are increase in guttate and erythrodermic psoriasis. HLA-B27 may be seen in pustular psoriasis. HLA-Cw2 is seen with late onset psoriasis, or type II. HLA-DR3 is commonly found with subacute cutaneous lupus Q/Q(M)-482152 Report a Problem Which enzyme can be a useful adjunct test to diagnose zinc deficiency where the serum zinc level is normal or near normal? 1

Zinc sulfatase

2

Alkaline phosphatase

3

AST

4

ALT

5

Creatinine kinase Q/Q(M)-477430 Report a Problem

Which enzyme can be a useful adjunct test to diagnose zinc deficiency where the serum zinc level is normal or near normal? 2

Alkaline phosphatase

A low serum alkaline phosphatase, a zinc dependent enzyme, may be a valuable adjuctive test where the serum zinc level is normal or near normal. The diagnosis of zinc deficiency should be suspected in at-risk individuals with acral or periorificial dermatitis. It is usually confirmed by low serum zinc levels. Q/Q(M)-477430 Report a Problem 59

This patient has similar lesions on his distal extremities. Which laboratory test can be done in order to make a diagnosis? 1

Hemogram

2

Alkaline phosphatase

3

Creatinine

4

Potassium

5

TSH Q/Q(M)-480486 Report a Problem

This patient has similar lesions on his distal extremities. Which laboratory test can be done in order to make a diagnosis? 2

Alkaline phosphatase

This patient has acrodermatitis enteropathica, a rare, inhertied disorder caused by an inability to absorb zinc. This disease is characterized by a traid of acral dermatitis, diarrhea, and alopecia. AE is rapidly reversed by zinc supplementation. Alkaline phosphatase is a zinc-dependent enzyme; it is a moderatelysensitive marker for zinc deficiency (although not an early marker). Q/Q(M)-480486 Report a Problem

Acute hemorrhagic edema of childhood is distinguished from Henoch-Schonlein Purpura based on: 1

The presence of pupura on the upper trunk

2

The lack of an antecedent infection

3

The involvement of the synovia

4

The neurologic complications

5

The lack of systemic features Q/Q(M)-480531 Report a Problem 60

Acute hemorrhagic edema of childhood is distinguished from Henoch-Schonlein Purpura based on: 5

The lack of systemic features

Acute hemorrhagic edema of childhood affects children and infants < 2 years of age. It presents with painful, edematous petechiae and ecchymoses on the head and distal extremities. Facial edema may be the initial sign. Triggering factors include infection, drugs, and immunization. It lacks the systemic features of HSP, and resolves in 1-3 weeks without sequelae. HSP occurs mostly in children. There is an antecedent URI in 75% of cases. HSP involves the skin, synovia, GI tract, and kidneys. Long-term morbidity results from renal disease, which is predicted by the spread of purpura to the upper trunk. Q/Q(M)-480531 Report a Problem

Which of the following is true about nevus anemicus: 1

Usually occurs in association with vitiligo

2

Occurs more frequently in men than in women

3

Most commonly involves the upper chest

4

Results from locally decreased vascular reactivity to catecholamines

5

Contains dilated blood vessels Q/Q(M)-481904 Report a Problem

Which of the following is true about nevus anemicus: 3

Most commonly involves the upper chest

Nevus anemicus is caused by localized hypersensitivity to catecholamines and most commonly found in the upper chest Q/Q(M)-481904 Report a Problem

A 32 year-old woman presents with moderate hirsutism. She has normal menses, normal-sized ovaries, no evidence of tumors of the adrenal or ovary, and normal adrenal function, but does have slight elevations of plasma androstenedione and testosterone. What is the most likely diagnosis? 1

Stein-Leventhal syndrome

2

Cushing syndrome

3

Idiopathic hirsutism

4

Occult virilizing tumor

5

Kruckenberg tumor Q/Q(M)-480499 Report a Problem

A 32 year-old woman presents with moderate hirsutism. She has normal menses, normal-sized ovaries, no evidence of tumors of the adrenal or ovary, and normal adrenal function, but does have slight elevations of plasma androstenedione and testosterone. What is the most likely diagnosis? 61

3

Idiopathic hirsutism

Idiopathic hirsutism is diagnosed in women with evidence of androgen excess but with normal menses, normal-sized ovaries, no evidence of tumors of the adrenal or ovary, and normal androgen function. Slight elevations of plasma androstenedione and testosterone are common. Q/Q(M)-480499 Report a Problem

Most common cause of these tender lesions on this 12-year old child is 1

Inflammatory bowel disease

2

Tuberculosis

3

Îbeta-hemolytic streptococcal infection

4

Yersinia

5

Infectious mononucleosis Q/Q(M)-482124 Report a Problem

Most common cause of these tender lesions on this 12-year old child is 3

Îbeta-hemolytic streptococcal infection

The picture shows classical case of erythema nodosum (EN). Although all other choices in question cause EN, the most common in the pediatric patient is beta-hemolytic streptococcal infection (especially pharyngitis). The management of erythema nodosum is directed at identification and treatment of the underlying cause. Minimal evaluation usually involves obtaining an ASO/DNase B titer, chest radiograph, and tuberculin testing. Bed rest, with elevation of the patient's legs, helps reduce pain and edema. When pain, inflammation, or arthralgia is prominent, NSAIDs can be prescribed. Salicylates, colchicine, and potassium iodide are the most commonly used alternative therapies. Q/Q(M)-482124 Report a Problem

This variant of amyloidosis is derived from degenerated tonofilaments of keratinocytes: 1

Lichen amyloidosis

2

Nodular amyloidosis

3

Primary systemic amyloidosis

4

Dialysis-related amyloidosis

5

Secondary systemic amyloidosis 62

Q/Q(M)-480476 Report a Problem

This variant of amyloidosis is derived from degenerated tonofilaments of keratinocytes: 1

Lichen amyloidosis

Lichen amyloidosis and Macular amyloidosis are derived from degenerated tonofilaments of keratinocytes. Primary systemic amyloidosis results from deposition of protein AL derived from Ig Lambda light chains. Nodular amyloidosis is also associated with AL type protein. Secondary systemic amyloidosis is associated with AA amyloid fibrils derived from SAA protein. Dialysis-related amyloidosis is associated with beta-2-microglobulin protein deposition. Q/Q(M)-480476 Report a Problem

Patients with chronic idiopathic urticaria should avoid: 1

Latex products

2

Alcohol

3

Nickel sulfate

4

�-blockers

5

aspirin Q/Q(M)-477167 Report a Problem

Patients with chronic idiopathic urticaria should avoid: 5

aspirin

Chronic idiopathic urticaria is defined by the presence of urticaria of unknown etiology lasting greater than 6 weeks. Patient with chronic idiopathic urticaria should avoid aspirin as it aggravates urticaria in about 30% of patients. Q/Q(M)-477167 Report a Problem Which of the following is a feature of Sneddon's syndrome? 1

Venous thromboses

2

Hepatosplenomegaly

3

anti-Scl-70 autoantibodies

4

M. tuberculosis infection

5

C2 deficiency Q/Q(M)-480535 Report a Problem

Which of the following is a feature of Sneddon's syndrome? 1

Venous thromboses

Sneddon's syndrome presents as livedo reticularis and livedoid vasculitis associated with ischemic 63

cerebrovascular lesions, hypotension, and extracerebral arterial and venous thromboses. Q/Q(M)-480535 Report a Problem

What organism caused this disease? 1

Micrococcus sedenditaris

2

Candida albicans

3

Staphylococcus aureus

4

Corynebacterium tenuis

5

Corynebacterium diptheria Q/Q(M)-482079 Report a Problem

What organism caused this disease? 1

Micrococcus sedenditaris

Pitted keratolysis is a skin disorder characterized by crateriform pitting that primarily affects the pressure-bearing aspects of the plantar surface of the feet. It is caused by a cutaneous infection with micrococcus sedentarius which is included in the Corynebacteria genus. These are gram-positive, catalase-positive, aerobic or facultatively anaerobic, generally nonmotile rods. Corynebacterium diphtheriae infection is classically characterized by a local inflammation, usually in the upper respiratory tract, associated with toxin-mediated cardiac and neural disease. Corynebacterium tenuis causes trichomycosis Patients typically present with yellow, black, or red pinpoint nodules on the hair shafts in the inguinal region. Staphylococcus aureus is a common bacterium that can result in formation of pustules, furuncles, cellulitis and abscesses. Candida species are responsible for superficial infections such as oropharyngeal candidiasis (thrush) and vulvovaginal candidiasis (vaginal Candidiasis) and is also occurs as an opportunistic infection Q/Q(M)-482079 Report a Problem

Acneiform eruptions have been associated with which of the following vitamins? 1

Vitamin C

2

Vitamin E

3

Vitamin A

4

Biotin

5

Vitamin B12 Q/Q(M)-474320 Report a Problem 64

Acneiform eruptions have been associated with which of the following vitamins? 5

Vitamin B12

Vitamin B12 can cause acneiform eruptions. Q/Q(M)-474320 Report a Problem

Which of the following is true regarding this diagnosis? 1

The protein deposited is derived from Ig light chains, kappa subtype

2

Skin is involved in
View more...

Comments

Copyright ©2017 KUPDF Inc.
SUPPORT KUPDF